Ableitung der Maxwell-Gleichungen aus dem Feldtensor-Lagrangian

Ich habe angefangen, Peskin und Schroeder in meiner Freizeit zu lesen, und ich bin etwas verwirrt darüber, wie man Maxwells Gleichungen aus der (quellenfreien) Lagrange-Dichte erhält L = 1 4 F μ v F μ v (wo F μ v = μ EIN v v EIN μ ist der Feldtensor).

Einsetzen in die Definition des Feldtensors liefert L = 1 2 [ ( μ EIN v ) ( μ EIN v ) ( μ EIN v ) ( v EIN μ ) ] . Ich weiß, ich sollte verwenden EIN μ als dynamische Variable in den Euler-Lagrange-Gleichungen, die werden L EIN μ μ L ( μ EIN v ) = μ L ( μ EIN v ) , aber ich bin verwirrt darüber, wie ich von hier aus fortfahren soll.

Ich weiß, ich sollte am Ende mit μ F μ v = 0 , aber ich verstehe nicht ganz warum. Seit μ und v Sind Dummy-Indizes, sollte ich sie ändern können: Wie verhalten sich die Indizes in der Lagrange-Funktion zu den Indizes in den Ableitungen in den Euler-Lagrange-Gleichungen?

Siehe in Sean Carolls Buch. Voller Ableitung dort
Warum reicht es nicht, die zu stecken F μ v = [ μ EIN v ] in Maxwells Gleichungen ein und zeigen, dass sie gelten?

Antworten (5)

Wir variieren die Aktion

δ L d t = δ Λ ( EIN v , μ EIN v ) d 3 x d t = 0
Λ ( EIN v , μ EIN v ) ist die Lagrangedichte des Systems.

So,

( Λ EIN v δ EIN v + Λ ( μ EIN v ) δ ( μ EIN v ) ) d 3 x d t = 0
Durch partielle Integration erhalten wir:
( Λ EIN v μ Λ ( μ EIN v ) ) δ EIN v d 3 x d t = 0 Λ EIN v μ Λ ( μ EIN v ) = 0
Wir müssen die Dichte des Lagrange bestimmen. Ein Begriff befasst sich mit der Wechselwirkung der Ladungen mit dem elektromagnetischen Feld, J μ EIN μ . Der andere Term ist die Energiedichte des elektromagnetischen Feldes: Dieser Term ist die Differenz des magnetischen Feldes und des elektrischen Feldes. Also haben wir:
Λ = J μ EIN μ + 1 4 μ 0 F μ v F μ v
Wir haben:
Λ EIN v = J v
Also:
μ Λ ( μ EIN v ) = 1 4 μ 0 μ ( ( μ EIN v ) F κ λ F κ λ ) = 1 4 μ 0 μ ( ( μ EIN v ) ( ( κ EIN λ λ EIN κ ) ( κ EIN λ λ EIN κ ) ) ) = 1 4 μ 0 μ ( ( μ EIN v ) ( κ EIN λ κ EIN λ κ EIN λ λ EIN κ λ EIN κ κ EIN λ + λ EIN κ λ EIN κ ) )
Der dritte und der vierte Term sind gleich dem ersten und dem zweiten Term. Du kannst tun k λ :
μ Λ ( μ EIN v ) = 1 2 μ 0 μ ( ( μ EIN v ) ( κ EIN λ κ EIN λ κ EIN λ λ EIN κ ) ) .
Aber
( μ EIN v ) ( κ EIN λ κ EIN λ ) = κ EIN λ ( μ EIN v ) ( κ EIN λ ) + κ EIN λ ( μ EIN v ) ( κ EIN λ ) = κ EIN λ δ κ μ δ λ v + g κ a g λ β κ EIN λ ( μ EIN v ) ( a EIN β ) = 2 μ EIN v .

Wir haben:

( μ EIN v ) ( κ EIN λ λ EIN κ ) = 2 v EIN μ .

So,

μ ( Λ ( μ EIN v ) ) = 1 μ 0 μ ( μ EIN v v EIN μ ) = 1 μ 0 μ F μ v .
Die Lagrange-Gleichungen liefern die nicht homogenen Maxwell-Gleichungen:

μ F μ v = μ 0 J v .

Zu Ihrer Information, diese Antwort (v4) verwendet implizit die Vorzeichenkonvention ( + , , , ) .

Nun, Sie haben es fast geschafft. Nutzen Sie die Tatsache, dass

( μ EIN v ) ( ρ EIN σ ) = δ μ ρ δ v σ
was gilt, weil μ EIN v sind d 2 unabhängige Komponenten.

Wie kann man das eigentlich beweisen?

Lieber Amc, schreiben Sie zuerst Ihre Lagrange-Dichte als

L = 1 4 F μ v F μ v = 1 2 ( μ EIN v ) F μ v
Ist das soweit in Ordnung? Das F μ v enthält zwei Terme, die es in den beiden Indizes antisymmetrisch machen. Es wird jedoch mit einem anderen multipliziert F μ v das ist bereits antisymmetrisch, also muss ich es nicht noch einmal antisymmetrisieren. Stattdessen geben mir beide Terme dasselbe, also den Koeffizienten 1 / 4 ändert sich einfach in 1 / 2 .

Jetzt zwingen Sie die Feldgleichungen, die Ableitungen der Lagrange-Funktion in Bezug auf zu berechnen EIN μ und seine Derivate. Zunächst einmal die Ableitung der Lagrange-Funktion L in Gedenken an EIN μ Komponenten selbst verschwindet, weil die Lagrange-Funktion nur von den partiellen Ableitungen von abhängt EIN μ . Ist das soweit klar?

Die Bewegungsgleichungen werden also sein

0 = μ [ L / ( μ EIN v ) ] =
Hoppla, Sie sind bereits an diesem Punkt angelangt. Aber schauen Sie sich jetzt meine Form der Lagrange-Funktion oben an. Die Ableitung der Lagrange-Funktion in Bezug auf μ EIN v ist einfach
1 2 F μ v
Weil μ EIN v einfach als Faktor erscheint, also werden die Bewegungsgleichungen einfach sein
0 = + 1 2 μ F μ v
Allerdings habe ich bewusst einen Fehler gemacht. Ich habe nur die Lagrangian in Bezug auf differenziert μ EIN v im ersten Faktor enthalten F μ v , mit den niedrigeren Indizes. Jedoch, μ EIN v Komponenten erscheinen auch in F μ v , der zweite Faktor in der Lagrange-Funktion, einer mit den oberen Indizes. Wenn Sie die entsprechenden Terme aus der Leibniz-Regel hinzufügen, ergibt sich einfach, dass sich der gesamte Beitrag verdoppelt. Die richtige Bewegungsgleichung, einschließlich des natürlichen Koeffizienten, wird also sein
0 = μ F μ v
Die Gesamtnormalisierung ist wichtig, da diese Gleichung zusätzliche Terme erhalten kann, wie z. B. den Strom, dessen Koeffizienten offensichtlich sind, und Sie möchten keinen relativen Fehler von zwei zwischen der Ableitung von erhalten F und der Strom j .

Gruß Lubos

Hey, ich weiß, das ist 5 Jahre zu spät, aber vielleicht siehst du das: Warum ( μ ϕ ) ( μ EIN v F μ v ) = F μ v . Hängt der Tensor nicht auch von den partiellen Ableitungen ab? Müssen wir dann nicht die Produktregel anwenden?
Hallo @ user17574 - hängt "welcher" Tensor nicht von den partiellen Ableitungen ab? Sicherlich tut das der Spannungs-Energie-Tensor und der Lagrange-Operator. Deshalb ist die Ableitung davon in Bezug auf die partiellen Ableitungen ungleich Null. Die Ableitung wird in der Antwort berechnet. Die Produktregel funktioniert tatsächlich und deshalb streicht man den Faktor von 1 / 2 . Hast du versucht, die Antwort zu lesen?
Ich weiß, das ist eine alte Frage, aber ich habe Zweifel. Nach Anwendung der Leibniz-Regel erhalten wir:
L ( μ EIN v ) = 1 2 ( μ EIN v F μ v ( μ EIN v ) + μ EIN v ( μ EIN v ) F μ v )
Wie soll ich das unterscheiden F μ v auf den ersten Term der Gleichung? Ich verstehe nicht, was mir das bringt F μ v .
Sie müssen ein neues Indexpaar verwenden, anstatt mu-nu dreimal zu kopieren. Dann F a β / ( γ EIN δ ) = g a γ g β δ a β einfach weil F ist nur der Unterschied zwischen zwei ähnlichen Termen (Antisymmetrisierung) und jeder Term hat die Ableitung, die im Grunde ein Kronecker-Delta ist - aber hier mit den erhöhten Indizes (um die obere Indexmetrik zu werden) wegen der Position der Indizes im ursprünglichen Ausdruck .

Obwohl ich spät in der Party bin, poste ich eine Antwort auf elementarer Ebene. Vielleicht beweist dies die Leistungsfähigkeit des Tensorkalküls, der in allen vorherigen netten Antworten verwendet wurde.

Abstrakt

In dieser Antwort werden wir versuchen, Maxwell-Gleichungen im leeren Raum abzuleiten

(001a) × E = B t (001b) × B = μ 0 j + 1 c 2 E t (001c) E = ρ ϵ 0 (001d) B = 0
aus den Euler-Lagrange-Gleichungen
(002) t ( L η ˙ ȷ ) + [ L ( η ȷ ) ] L η ȷ = 0 , ( ȷ = 1 , 2 , 3 , 4 )
wo
(003) L = L ( η ȷ , η ˙ ȷ , η ȷ ) ( ȷ = 1 , 2 , 3 , 4 )
ist die Lagrange-Dichte der Frage (außer einem konstanten Faktor)
(004) L = E 2 c 2 B 2 2 + 1 ϵ 0 ( ρ ϕ + j EIN )
und η ȷ ( x 1 , x 2 , x 3 , t ) , ȷ = 1 , 2 , 3 , 4 die Komponenten EIN 1 , EIN 2 , EIN 3 , ϕ des EM-Potenzial-4-Vektors. In gewissem Sinne baut diese Ableitung auf der Umkehrung auf (: das Finden einer richtigen Lagrange-Dichte aus Maxwell-Gleichungen ), indem man sich rückwärts bewegt, siehe meine Antwort hier: Ableitung der Lagrange-Dichte für elektromagnetisches Feld

1. Hauptabschnitt

Zuerst drücken wir aus E , B von (004) in Bezug auf die potentiellen 4-Vektor-Komponenten EIN 1 , EIN 2 , EIN 3 , ϕ

(005a) B = × EIN (005b) E = ϕ EIN t = ϕ EIN ˙
Ab (005) gelten automatisch die Maxwell-Gleichungen (001a) und (001d). Die vier(4) skalaren Maxwell-Gleichungen (001b) und (001c) müssen also aus den vier(4) skalaren Euler-Lagrange-Gleichungen (002) abgeleitet werden. Außerdem ist es vernünftig anzunehmen, dass die Vektorgleichung (001b) in Bezug auf die Komponenten des Vektorpotentials von (002) abgeleitet werden muss EIN = ( EIN 1 , EIN 2 , EIN 3 ) , während die skalare Gleichung (001c) bezüglich des skalaren Potentials aus (002) abgeleitet werden muss ϕ .

Aus den Gleichungen (005) drücken wir die Lagrange-Dichte (004) in Form der potentiellen 4-Vektor-Komponenten aus EIN 1 , EIN 2 , EIN 3 , ϕ :

(006a) E 2 = ϕ EIN t 2 = EIN ˙ 2 + ϕ 2 + 2 ( ϕ EIN ˙ ) (006b) B 2 = × EIN 2 k = 1 k = 3 [ EIN k 2 EIN x k EIN k ]
Die zweite Gleichung in (006b), das ist die Identität
(Id-01) × EIN 2 k = 1 k = 3 [ EIN k 2 EIN x k EIN k ]
wird in 2. Identitäten Abschnitt bewiesen . Einfügen von Ausdrücken (006) in (004) ist die Lagrange-Dichte
(007) L = 1 2 EIN ˙ 2 + 1 2 ϕ 2 + ϕ EIN ˙ 1 2 ϕ EIN t 2 1 2 c 2 k = 1 k = 3 [ EIN k 2 EIN x k EIN k ] × EIN 2 + 1 ϵ 0 ( ρ ϕ + j EIN )

Wir ordnen die Items in (007) wie folgt neu:

(008a) L = 1 2 ϕ 2 ρ ϕ ϵ 0 + ϕ EIN ˙ L ϕ = in Gedenken an  ϕ + 1 2 EIN ˙ 2 + 1 2 c 2 k = 1 k = 3 [ EIN x k EIN k EIN k 2 ] + j EIN ϵ 0 (008b) L = 1 2 ϕ 2 ρ ϕ ϵ 0 + ϕ EIN ˙ + 1 2 EIN ˙ 2 + 1 2 c 2 k = 1 k = 3 [ EIN x k EIN k EIN k 2 ] + j EIN ϵ 0 L EIN = in Gedenken an  EIN

Das L ϕ ein Teil der Dichte enthält alles ϕ -Bedingungen und wird vernünftigerweise allein auf die Ableitung der Maxwell-Gleichung (001c) aus der Euler-Lagrange-Gleichung (002) bezogen η 4 = ϕ . Das L EIN ein Teil der Dichte enthält alles EIN -Bedingungen und wird vernünftigerweise allein auf die Ableitung der Maxwell-Gleichung (001b) aus den Euler-Lagrange-Gleichungen (002) bezogen η 1 , η 2 , η 3 = EIN 1 , EIN 1 , EIN 3 . Beachten Sie den gemeinsamen Begriff ϕ EIN ˙ der Teile L ϕ , L EIN .

Die Euler-Lagrange-Gleichung bzgl η 4 = ϕ ist :

(009) t ( L ϕ ˙ ) 0 + [ L ( ϕ ) ] ϕ + EIN ˙ L ϕ ρ ϵ 0 = 0
oder
(010) ( ϕ EIN t ) E = ρ ϵ 0
das ist die Maxwell-Gleichung (001c)
(001c) E = ρ ϵ 0

Um die Maxwell-Gleichung (001b) herzuleiten, drücken wir sie mit Hilfe der Gleichungen (005) durch die potentiellen 4-Vektor-Komponenten aus EIN 1 , EIN 2 , EIN 3 , ϕ :

(011) × ( × EIN ) = μ 0 j + 1 c 2 t ( ϕ EIN t )
Verwendung der Identität
(012) × ( × EIN ) = ( EIN ) 2 EIN
Gl. (011) liefert
(013) 1 c 2 2 EIN t 2 2 EIN + ( EIN + 1 c 2 ϕ t ) = μ 0 j
Das k -Komponente von Gl. (013) wird korrekt ausgedrückt, um wie folgt wie eine Euler-Lagrange-Gleichung auszusehen:
(014) t ( EIN k t + ϕ x k ) + [ c 2 ( EIN x k EIN k ) ] j k ϵ 0 = 0
Es genügt, über Gl. (014) aus der Euler-Lagrange-Gleichung (002) bzgl η k = EIN k , k = 1 , 2 , 3 :

(015) t ( L EIN ˙ k ) + [ L ( EIN k ) ] L EIN k = 0

Jetzt

(016a) L EIN ˙ k = EIN ˙ k ( ϕ EIN ˙ + 1 2 EIN ˙ 2 ) = ϕ x k + EIN k t

(016b) L EIN k = EIN k ( j EIN ϵ 0 ) = j k ϵ 0
und
(016c) L ( EIN k ) = ( EIN k ) ( 1 2 c 2 k = 1 k = 3 [ EIN x k EIN k EIN k 2 ] ) = c 2 ( EIN x k EIN k )
Die letzte Gleichung in (016c) ist wegen der Identität (Id-02) gültig, die in 2. Identitätsabschnitt bewiesen wurde :
(Id-02) ( | | × EIN | | 2 ) ( EIN k ) = ( EIN k ) ( k = 1 k = 3 [ EIN x k EIN k EIN k 2 ] ) = 2 ( EIN k EIN x k )
Unter Verwendung der Ausdrücke der Gleichungen (016) ergibt die Euler-Lagrange-Gleichung (015) (014) und somit die Maxwell-Gleichung (001b).

2. Abschnitt Identitäten

Wenn EIN = ( EIN 1 , EIN 2 , EIN 3 ) ist eine Vektorfunktion der kartesischen Koordinaten ( x 1 , x 2 , x 3 ) dann

(Id-01) × EIN 2 k = 1 k = 3 [ EIN k 2 EIN x k EIN k ]
und
(Id-02) ( | | × EIN | | 2 ) ( EIN k ) = ( EIN k ) ( k = 1 k = 3 [ EIN x k EIN k EIN k 2 ] ) = 2 ( EIN k EIN x k )
wobei die funktionale Ableitung der linken Seite definiert ist als
(Id-03) ( | | × EIN | | 2 ) ( EIN k ) [ ( | | × EIN | | 2 ) ( EIN k x 1 ) , ( | | × EIN | | 2 ) ( EIN k x 2 ) , ( | | × EIN | | 2 ) ( EIN k x 3 ) ]
Beweis der Gleichung (Id-01) :
| | × EIN | | 2 = ( EIN 3 x 2 EIN 2 x 3 ) 2 + ( EIN 1 x 3 EIN 3 x 1 ) 2 + ( EIN 2 x 1 EIN 1 x 2 ) 2 = [ ( EIN 1 x 2 ) 2 + ( EIN 1 x 3 ) 2 ] + [ ( EIN 2 x 1 ) 2 + ( EIN 2 x 3 ) 2 ] + [ ( EIN 3 x 1 ) 2 + ( EIN 3 x 2 ) 2 ] 2 [ EIN 1 x 2 EIN 2 x 1 + EIN 2 x 3 EIN 3 x 2 + EIN 3 x 1 EIN 1 x 3 ] = [ ( EIN 1 x 1 ) 2 + ( EIN 1 x 2 ) 2 + ( EIN 1 x 3 ) 2 ] + [ ( EIN 2 x 1 ) 2 + ( EIN 2 x 2 ) 2 + ( EIN 2 x 3 ) 2 ] + [ ( EIN 3 x 1 ) 2 + ( EIN 3 x 2 ) 2 + ( EIN 3 x 3 ) 2 ] [ ( EIN 1 x 1 ) 2 + ( EIN 2 x 2 ) 2 + ( EIN 3 x 3 ) 2 ] 2 [ EIN 1 x 2 EIN 2 x 1 + EIN 2 x 3 EIN 3 x 2 + EIN 3 x 1 EIN 1 x 3 ] = EIN 1 2 + EIN 2 2 + EIN 3 2 ( EIN 1 x 1 EIN 1 x 1 + EIN 2 x 1 EIN 1 x 2 + EIN 3 x 1 EIN 1 x 3 ) ( EIN 1 x 2 EIN 2 x 1 + EIN 2 x 2 EIN 2 x 2 + EIN 3 x 2 EIN 2 x 3 ) ( EIN 1 x 3 EIN 3 x 1 + EIN 2 x 3 EIN 3 x 2 + EIN 3 x 3 EIN 3 x 3 ) = EIN 1 2 + EIN 2 2 + EIN 3 2 EIN x 1 EIN 1 EIN x 2 EIN 2 EIN x 3 EIN 3 = k = 1 k = 3 [ EIN k 2 EIN x k EIN k ]
Beweis der Gleichung (Id-02): Aus Gleichung
| | × EIN | | 2 = ( EIN 3 x 2 EIN 2 x 3 ) 2 + ( EIN 1 x 3 EIN 3 x 1 ) 2 + ( EIN 2 x 1 EIN 1 x 2 ) 2 = [ ( EIN 1 x 2 ) 2 + ( EIN 1 x 3 ) 2 ] + [ ( EIN 2 x 1 ) 2 + ( EIN 2 x 3 ) 2 ] + [ ( EIN 3 x 1 ) 2 + ( EIN 3 x 2 ) 2 ] 2 [ EIN 1 x 2 EIN 2 x 1 + EIN 2 x 3 EIN 3 x 2 + EIN 3 x 1 EIN 1 x 3 ]
wir haben
( | | × EIN | | 2 ) ( EIN 1 x 1 ) = 0 = 2 ( EIN 1 x 1 EIN 1 x 1 ) ( | | × EIN | | 2 ) ( EIN 1 x 2 ) = 2 ( EIN 1 x 2 EIN 2 x 1 ) ( | | × EIN | | 2 ) ( EIN 1 x 3 ) = 2 ( EIN 1 x 3 EIN 3 x 1 )
So
( | | × EIN | | 2 ) ( EIN 1 ) = 2 ( EIN 1 EIN x 1 )
Beweisgleichung (Id-02) für k = 1 und ähnlich für die anderen beiden Komponenten k = 2 , 3 .

Eine Methode besteht darin, die Maxwell-Aktion (set J μ = 0 wenn Sie wollen, für den quellenfreien Fall)

S = d 4 x L = d 4 x ( 1 4 F μ v F μ v + J μ EIN μ ) .
Notieren Sie sich das zunächst
δ ( F μ v F μ v ) = 2 F μ v δ F μ v = 2 F μ v ( μ δ EIN v v δ EIN μ ) = 4 F μ v μ δ EIN v = 4 [ μ ( F μ v δ EIN v ) μ F μ v δ EIN v ] ,
wo wir die Tatsache verwendet haben, dass F ist antisymmetrisch.

Beachten Sie auch, dass die μ ( F μ v δ EIN v ) Der Begriff verschwindet, wenn er mit dem Standardargument that in ein Oberflächenintegral umgewandelt wird δ EIN μ verschwindet an der Integrationsgrenze.

Unter Verwendung des Obigen ist die Variation der Aktion

δ S = d 4 x   δ EIN v ( μ F μ v + J v ) ,
was seit δ EIN v willkürlich ist, führt zum gewünschten Ergebnis
μ F μ v = J v .

Anstatt zu vermuten δ EIN μ Verschwinden an der Grenze, von der man ausgehen kann F μ v an der Grenze. Ist das falsch? Siehe eine verwandte Frage hier physical.stackexchange.com/questions/438277/… @EricAngle